LSAT and Law School Admissions Forum

Get expert LSAT preparation and law school admissions advice from PowerScore Test Preparation.

 Administrator
PowerScore Staff
  • PowerScore Staff
  • Posts: 8916
  • Joined: Feb 02, 2011
|
#36814
Complete Question Explanation

Main Point. The correct answer choice is (E)

In this stimulus, the sociologist concludes that the supporters of political democracy can also support
marketplace regulation. This conclusion, which comes at the end of the stimulus and is introduced with
the word “hence,” is based on the premise that there is a crucial distinction between the private consumer
and the public citizen: Private consumers choose based on self-interest, argues the author, but the public
citizen decides on the basis of beliefs about what is best for society.
In this case the question asks for the conclusion, which we should prephrase: supporters of political
democracy can also support marketplace regulation.

Answer choice (A): This is perhaps an implication found in the stimulus, but it is not the conclusion, so
this choice is incorrect.

Answer choice (B): The author’s conclusion is that regulated markets are compatible with democratic
sovereignty, not that unregulated markets are incompatible. This answer choice is basically a Mistaken
Negation and is therefore incorrect.

Answer choice (C): This answer choice specifically reflects the belief of “some economists,” as
described in the stimulus, which is a position the author attacks. This choice is thus incorrect.

Answer choice (D): This is a premise used to support the conclusion, it is not the conclusion itself, so
this answer choice is incorrect.

Answer choice (E): This is the correct answer choice. It is basically a rephrasing of the conclusion,
found at the end of the stimulus.
 desmail
  • Posts: 50
  • Joined: Jul 05, 2011
|
#3744
Hi,

I'm really confused with this question and I'm having trouble understanding what the author is trying to say in the stimulus. Is the conclusion in the last sentence, "hence..?"

My problem with answer choice E is that the author is just saying that democracy could support market regulation, but this doesn’t necessarily mean there is an “opposition” to unregulated ones. And how do we know that it is always "consistent" with support for democracy? It just seems too strong of an answer.

What is wrong with answer choices B and D?

Thank you!
Dana
 Nikki Siclunov
PowerScore Staff
  • PowerScore Staff
  • Posts: 1362
  • Joined: Aug 02, 2011
|
#3748
Hi Dana,

The conclusion is indeed the last sentence, but to understand it better, try to go back to the beginning. Whenever the argument begins by outlining someone else's position, the author's conclusion is likely to counter that position.

To simplify this, the economists hold that unregulated markets should accompany democracy. The author disagrees, holding that people can support democracy while also supporting marketplace regulation. In other words, support for democracy is consistent with support for marketplace regulation. Answer choice (E) agrees with this prephrase, even though it uses the double negative (opposition to unregulated markets = support for regulated markets).

Answer choice (B) is too strong. Just because democratic sovereignty is compatible with marketplace regulation does not mean that unregulated markets are incompatible with it.

Answer choice (D) is a premise, not the conclusion of the argument.

Hope this helps!
 desmail
  • Posts: 50
  • Joined: Jul 05, 2011
|
#3749
Hi Nikki,

But if something is "consistent" with support for democracy, doesn't that mean that the two things occur together most of the time? The author is just saying that supporters of democracy "can" support market regulation, in that it is possible, but it doesn't mean it will happen most of the time. I just cant see how they would have translated our conclusion in the stimulus to answer (E).

Also, do you know where can I learn more about double negatives? (for future practice)

I really would appreciate your help!
Dana
 Nikki Siclunov
PowerScore Staff
  • PowerScore Staff
  • Posts: 1362
  • Joined: Aug 02, 2011
|
#3753
Hi Dana,

I don't believe that the word "consistent" has the exact meaning you ascribe to it.

"Consistent" is an adjective with two meanings:

1. (of a person, behavior, or process) Unchanging in achievement or effect over a period of time: "the president is a consistent opponent to nuclear disarmament." This is not the meaning we're dealing with here.

2. Compatible or in agreement with something: "injuries consistent with falling". You don't get injuries every time you fall, or fall every time you got injured. But it is possible (i.e. it "could be true") that you got injured from falling. This is the meaning most commonly used on the test.

Unfortunately, we don't have a list of questions with double negatives, mostly because there are just so many of them. One that comes to mind is Q. 18 in October 2008 LR1.

Good luck!
 chian9010
  • Posts: 81
  • Joined: Jun 08, 2018
|
#58939
Hi All,

I understand why E is correct but I have a question for B.

The claim of some economists is that "unregulated markets should accompany democratic sovereignty because they let people vote with their money." However, it seems like the sociologist is trying to attack/against their claim. Therefore, if we negate this claim, will it become sociologist's claim? If so, then can we say the sociologist's claim is "unregulated markets should NOT accompany democratic sovereignty" It seems to me, B is the analogy of the negated claim.

OR even if the sociologist does not agree with some economists' claim, we can not say the negated claim is the sociologist's claim?
 Brook Miscoski
PowerScore Staff
  • PowerScore Staff
  • Posts: 418
  • Joined: Sep 13, 2018
|
#60937
Chian,

It is good that you see that (E) is the most direct expression of the conclusion of the sociologist, which is that it is possible to support both democracy and regulation (see last sentence, "can also"). That also explains why (B) does not express the sociologist's conclusion. The sociologist doesn't say that people who support democracy must support regulation, just that they can also support regulation.

Similarly, (B) is not a correct negation of the "some economists" position. "Some economists" believe that people who support democracy should oppose regulation. The correct negation is that people who support democracy do not need to oppose regulation. When negating, construct logical opposites (not polar opposites)!!! What you are negating is the idea that something is favored, the logical opposite is neutrality, not opposition.

Get the most out of your LSAT Prep Plus subscription.

Analyze and track your performance with our Testing and Analytics Package.